Jump to content
BrainDen.com - Brain Teasers
  • 0


Guest
 Share

Question

4 answers to this question

Recommended Posts

  • 0

Find the value of P in the following infinite product:

x^3 + 1 = (x+1)(x^2-x+1)

x^3 - 1 = (x-1)(x^2+x+1)

f(k) = (k - 1)*(kk + k + 1)

g(k) = (k + 1)*(kk - k + 1)

each term t(k) = f(k) / g(k)

we start with k=2, or 7/9 and proceed infinitely.

P = product(k = 2 to infinity) of t(k)

so we have for k=2:

f(k)/g(k) * f(k+1)/g(k+1) * f(k+2)/g(k+2) * f(k+3)/g(k+3) *...

expanding the first two fractions we get:

t(k) * t(k+1) = (k - 1)*(kk + k + 1) / (k + 1)*(kk - k + 1) * (k+1 - 1)*((k+1)^2 + k+1 + 1) / (k+1 + 1)*((k+1)^2 - k -1+ 1) * ....

= (k - 1)*(kk + k + 1) / (k + 1)(kk - k + 1) * (k)(kk + 2k + 1 + k + 1 + 1) / (k+2)(kk + 2k + 1 - k - 1 + 1)

= (k - 1)*(kk + k + 1) / (k + 1)(kk - k + 1) * (k)(kk + 3k + 3) / (k+2)(kk + k + 1)

= (k-1)(k) (kk + k + 1) (kk + 3k + 3) / [ (k+1)(kk - k + 1)(k+2)(kk + k + 1) ]

= (k-1)(k) / (k+1)(k+2) * (kk + 3k + 3) / (kk - k + 1)

= 1/6 (4 +6 + 3)/(4-2+1)

= 13/18 in the case of 2, which matches so far t(2) * t(3)

so let's say h(k) = t(k) * t(k+1)

now jump forward by two k's. The new h(k) has the same opening thing: (k-1)(k) / (k+1)(k+2)

except here, the (k-1) and (k) is equal to the (k+1) and (k+2) of the previous h(k), and therefore cancels out, leaving only the initial (k-1)(k) on that far side which is (1)(2) = 2.

As for the second half of the h(k) formula, let's apply h(k+2)

(kk + 4k + 4 + 3k + 6 + 3) / (kk + 4k + 4 - k - 2 + 1) = (kk + 7k + 13) / (kk + 3k + 3)

thus the (kk + 3k + 3) cancels out, etc. Each new term cancelling the one before. Leaving, for this half of the equation, only the original divison by (kk - k + 1) which is (4 - 2 + 1) = 3

so we're left with (2)/(3) = 2/3

there's probably a way easier way to do this haha

Link to comment
Share on other sites

  • 0

Find the value of P in the following infinite product:

Factorizing numerator and denominator, we have

k3 − 1 = (k − 1)(k2 + k + 1)

k3 + 1 = (k + 1)(k2 − k + 1)

Note that k2 − k + 1 = (k − 1)2 + (k − 1) + 1, and so k3 + 1 = [(k − 1) + 2][(k − 1)2 + (k − 1) + 1], allowing cancellation of the quadratic factor across successive terms, and of the linear factor across "next but one" terms.

We can now calculate Pn, the partial product of the first n − 1 terms.

P(n) = 7/9 * 26/28 * 63/65 * ... * (n^3 - 1)/(n^3 + 1) = (1/3 * 7/3) * (2/4 * 13/7) * (3/5 * 21/13) * ... * [(n-1)/(n+1) * (n^2+n+1)/(n^2-n+1)] = (2/3) * [(n^2+n+1)/(n(n+1))] = (2/3) * [1 + 1/(n(n+1))]

As n tends to infinity, Pn tends to 2/3.

That is, the infinite product, P, converges to 2/3; P = Pinfinity = 2/3.

Link to comment
Share on other sites

Join the conversation

You can post now and register later. If you have an account, sign in now to post with your account.

Guest
Answer this question...

×   Pasted as rich text.   Paste as plain text instead

  Only 75 emoji are allowed.

×   Your link has been automatically embedded.   Display as a link instead

×   Your previous content has been restored.   Clear editor

×   You cannot paste images directly. Upload or insert images from URL.

Loading...
 Share

  • Recently Browsing   0 members

    • No registered users viewing this page.
×
×
  • Create New...